Integrale triplo coordinate cilindriche

Messaggioda Albi_b7 » 29/08/2014, 15:26

Mi sono trovato a dover risolvere il seguente integrale triplo:

$ int_(A) 1/(1+z^2)dxdydz $

Dove A è definito come:

$ A={(x,y,z)inR^3|x>=0,y>=0,sqrt(x^2+y^2)<=z<=(2-(x^2+y^2))} $

Avevo pensato di procedere tramite coordinate cilindriche ottenendo i seguenti estremi di integrazione:

$ rho<=z<=(2-rho^2) $
$ 0<=rho<=2 $
$ 0<=vartheta<=pi/2 $

non avendo il risultato non so se il mio integrale risulta giusto (oltretutto vengono calcoli importanti) volevo solo sapere se ho calcolato bene gli estremi di integrazione, essendo l'unica cosa che ormai mi crea difficoltà negli integrali.
Albi_b7
Starting Member
Starting Member
 
Messaggio: 4 di 16
Iscritto il: 29/08/2014, 10:37

Re: Integrale triplo coordinate cilindriche

Messaggioda ciampax » 29/08/2014, 19:56

Non mi torna la $\rho$. Per determinare le limitazioni per $z$ e $\rho$ puoi disegnare le curve $z=\rho$ (retta) e $z=2-\rho^2$ (parabola con asse coincidente con l'asse $z$ e rivolta verso il basso) nel piano $\rho O z$ tenendo conto solo del primo e del quarto quadrante (dove $\rho\ge 0$). Le due curve si intersecano nel punto $(1,1)$ (anche nel punto $(-2,-2)$ ma lo escludiamo per ché $\rho=-2$ non è accettabile). Di conseguenza si ha
$$\theta\in[0,\pi/2],\quad \rho\in[0,1],\quad \rho\le z\le 2-\rho^2$$
Il problema effettivamente sono i calcoli, visto che si ha
$$\int_0^{\pi/2}\int_0^1\int_\rho^{2-\rho^2}\frac{\rho}{1+z^2}\ dz\ d\rho\ d\theta=\frac{\pi}{2}\int_0^1\left[\rho\arctan(2-\rho^2)-\rho\arctan\rho\right]\ d\rho$$
dove l'ultimo integrale va risolto per parti e porta fuori una cosa del genere
$$\frac{\pi}{4}\int_0^1 \rho^2\left[\frac{2\rho}{1+(2-\rho^2)^2}+\frac{1}{1+\rho^2}\right]\ d\rho$$
Ultima modifica di ciampax il 30/08/2014, 10:13, modificato 1 volta in totale.
ciampax
Cannot live without
Cannot live without
 
Messaggio: 6709 di 14122
Iscritto il: 16/03/2009, 21:13

Re: Integrale triplo coordinate cilindriche

Messaggioda Albi_b7 » 30/08/2014, 09:00

Ecco, ho sempre dei problemi nel determinare gli estremi di integrazione. Non è che potresti darmi alcuni consigli o, eventualmente, dei passaggi da eseguire (sarebbe oro) per determinarli? Ormai non so piu dove sbattere la testa, purtroppo quando un professore non è appassionato nella sua materia non è mai un aiuto per i suoi studenti.
Albi_b7
Starting Member
Starting Member
 
Messaggio: 5 di 16
Iscritto il: 29/08/2014, 10:37

Re: Integrale triplo coordinate cilindriche

Messaggioda ciampax » 30/08/2014, 10:35

Ho modificato sopra: avevo dimenticato la $\rho$ dello Jacobiano.
Per quanto riguarda la tua richiesta, il consiglio è quello di farsi un buon disegno: di solito, il passaggio a nuove coordinate permette sempre di realizzare grafici di curve dipendenti da solo due delle nuove coordinate, o, nel caso siano presenti ancora tutte e tre, è possibile che una di esse funzioni come un parametro.

Per darti un idea, pensa di voler calcolare un integrale triplo su un dominio limitato dalle due superfici
$$x^2+2y^2+z^2=4,\qquad z-x^2-2y^2=1$$
Passiamo a coordinate cilindriche: per prima cosa osservi che le due superfici sono di rotazione attorno all'asse $z$ e questo ti permette di affermare che $\theta\in[0,2\pi]$. Effettuando il cambiamento di coordinate si ha
$$z^2+\rho^2 G(\theta)=4,\qquad z-\rho^2 G(\theta)=1$$
dove ho posto
$$G(\theta)=\cos^2\theta+2\sin^2\theta=1+\sin^2\theta>0$$
Dal momento che, per ogni $\theta$ fissato, abbiamo le due curve nelle coordinate $\rho, z$ scritte sopra, puoi andare di nuovo a fare un disegno di queste nel piano come ho fatto prima. Le due curve sono, rispettivamente, una ellisse di semiassi $2/\sqrt{G(\theta)}$ e $2$, e una parabola con asse coincidente con l'asse $z$, vertice in $(0,1)$ e concavità rivolta verso l'alto. Disegnandole, si vede che l'ellisse "contiene" un pezzo di parabola, a prescindere del valore di $G$, e quindi la limitazione necessaria per $z$ sarà una cosa del tipo "parabola" sotto $z$ sotto "ellisse". Intersecando le curve si trova l'unico punto di intersezione accettabile $\rho=\sqrt{\frac{(\sqrt{21}-3)}{2 G(\theta)}}$ e quindi si conclude che le limitazioni sono le seguenti
$$\theta\in[0,2\pi],\quad\rho\in\left[0,\sqrt{\frac{(\sqrt{21}-3)}{2 G(\theta)}}\right],\quad 1+\rho^2 G(\theta)\le z\le\sqrt{4-\rho^2 G(\theta)}$$
con l'integrale che va svolto nell'ordine: $z\to \rho\to\theta$.
ciampax
Cannot live without
Cannot live without
 
Messaggio: 6714 di 14122
Iscritto il: 16/03/2009, 21:13


Torna a Analisi matematica di base

Chi c’è in linea

Visitano il forum: Nessuno e 1 ospite